Wie kann man beweisen, dass die Umkehrung einer stetig streng monoton steigenden Funktion stetig ist? (Terence-Tao-Analyse 1, Satz 9.8.3)

Ich habe einige Probleme zu beweisen, dass die Inverse stetig ist. Der Hinweis in dem Buch ist, die Standard-Epsilon-Delta-Definition von Kontinuität zu verwenden. Ich glaube, der einfachste Weg ist ein Beweis durch Widerspruch, aber mit all den Quantifizierern in der Aussage negiere ich möglicherweise fälschlicherweise die Aussage, die ich zu beweisen versuche. Außerdem steht mir der Zwischenwertsatz zur Verfügung, auf dem sich die meisten meiner Beweise stützen. Nachfolgend der Vorschlag:

Lassen A < B reelle Zahlen sein, und lassen F : [ A , B ] R eine Funktion sein, die sowohl stetig als auch streng monoton steigend ist. Dann F ist eine Bijektion von [ A , B ] Zu [ F ( A ) , F ( B ) ] , und die Umkehrung F 1 : [ F ( A ) , F ( B ) ] [ A , B ] ist ebenfalls stetig und streng monoton steigend.

Nachfolgend mein Beweisversuch:

Lassen X 1 , X 2 [ A , B ] reelle Zahlen sein, so dass F ( X 1 ) = F ( X 2 ) . Aus der Trichotomie der reellen Zahlen haben wir, dass genau eine der folgenden Aussagen zutrifft: X 1 = X 2 , X 1 < X 2 , oder X 1 > X 2 . Vermuten X 1 X 2 . Dann haben wir das per Definition streng steigender monotoner Funktionen F ( X 1 ) F ( X 2 ) . Daher, X 1 = X 2 , Und F ist injektiv. Nun lass j [ F ( A ) , F ( B ) ] eine reelle Zahl sein. Dann existiert nach dem Zwischenwertsatz eine reelle Zahl C [ A , B ] so dass F ( C ) = j . Daher, F ist eine Surjektion aus [ A , B ] Zu [ F ( A ) , F ( B ) ] . Seit F sowohl injektiv als auch surjektiv ist, können wir daraus schließen F ist eine Bijektion von [ A , B ] Zu [ F ( A ) , F ( B ) ] . Zu zeigen, dass F 1 ist streng monoton steigend, let j 1 , j 2 [ F ( A ) , F ( B ) ] reelle Zahlen sein, so dass j 1 < j 2 . Dann existieren nach dem Zwischenwertsatz X 1 , X 2 [ A , B ] so dass F ( X 1 ) = j 1 Und F ( X 2 ) = j 2 . Seit F streng monoton steigend ist, haben wir X 1 < X 2 . Unter Verwendung der Definition einer Inversen haben wir

F 1 ( j 1 ) = F 1 ( F ( X 1 ) ) = X 1 < X 2 = F 1 ( F ( X 2 ) ) = F 1 ( j 2 ) ,
Zeigt das F 1 ist streng monoton steigend. Abschließend werden wir das zeigen F 1 ist kontinuierlich. Lassen j 0 [ F ( A ) , F ( B ) ] eine reelle Zahl sein, und lassen ϵ > 0 eine reelle Zahl sein. Wie zuvor existiert eine reelle Zahl X 0 [ A , B ] so dass F ( X 0 ) = j 0 . Ebenso für jede reelle Zahl j [ F ( A ) , F ( B ) ] , sagt uns der Zwischenwertsatz, dass es eine reelle Zahl gibt X [ A , B ] so dass F ( X ) = j . Wir wollen zeigen, dass es a gibt δ > 0 so dass | F 1 ( j ) F 1 ( j 0 ) | < ϵ für alle j [ F ( A ) , F ( B ) ] so dass | j j 0 | < δ . Dies ist gleichbedeutend mit dem Beweis, dass es a gibt δ > 0 so dass | X X 0 | < ϵ für alle F ( X ) [ F ( A ) , F ( B ) ] so dass | F ( X ) F ( X 0 ) | < δ . In der Reihenfolge geschrieben, an die wir eher gewöhnt sind, ist dies gleichbedeutend damit, zu zeigen, dass es a gibt δ > 0 so dass | F ( X ) F ( X 0 ) | < δ für alle X [ A , B ] so dass | X X 0 | < ϵ . Nehmen wir aus Gründen des Widerspruchs an, dass F 1 ist nicht kontinuierlich. Das heißt, angenommen für alle δ > 0 , es existiert ein ϵ > 0 so dass | F ( X ) F ( X 0 ) | δ für alle X [ A , B ] so dass | X X 0 | < ϵ .

Ich bin mir nicht wirklich sicher, wohin ich von hier aus gehen soll, und ich bin mir nicht sicher, ob ich die Aussage richtig verneint habe, dass die Umkehrung von F ist kontinuierlich. Jede Hilfe wird sehr geschätzt.

PS Dies ist keine Hausaufgabe, sondern nur das Selbststudium. Ich habe noch nie einen Kurs in Analyse belegt, also fühlen Sie sich frei, auf etwas hinzuweisen, was ich falsch mache (oder das weniger als streng ist).

Lass mich das klarstellen. Sie versuchen nicht zu beweisen, dass die Umkehrung einer stetig monoton ansteigenden Funktion stetig ist , Sie versuchen zu beweisen, dass sie eine stetige Umkehrung hat ? Ist das richtig? Sie haben eine kontinuierlich monoton ansteigende Funktion F und Sie wollen beweisen, dass die Umkehrung von F hat eine stetige Inverse?
Kurze Anmerkung zu Ihrer Antwort: Sie haben das Bild angenommen F Ist [ F ( A ) , F ( B ) ] aber das war keine Annahme in der Frage. Es ist wahr, aber technisch muss man es beweisen. Um den Stetigkeitsbeweis zu vereinfachen, beachten Sie die Definition von stetig auf Punkt A ist dasselbe wie: für alle ϵ > 0 es existiert δ st wenn X ( A δ , A + δ ) Dann F ( X ) ( F ( A ) ϵ , F ( A ) + ϵ ) . Hinweis: Diese Funktion und ihre Umkehrung bilden Intervalle auf Intervalle ab
@MBW Ich glaube, das ist eine Folge des Zwischenwertsatzes, der auch hier angenommen zu werden scheint. Beides muss nachgewiesen werden.
@bof, ich versuche das zu beweisen F hat eine stetige Inverse. Und es nimmt streng zu (wie Sie bemerkt haben, ist dies notwendig für F injektiv sein).
@MBW, Sie haben Recht, dass ich das beweisen müsste. Das ist eine logische Folge der IVT, und ich habe es bewiesen, bevor ich zu diesem Punkt kam (also hätte ich sagen sollen, dass es eine unserer Annahmen ist).
@bof mein Fehler. Werde das jetzt ändern.

Antworten (1)

Verwenden Sie diese Formulierung des Problems:

Für alle j 0 ( F ( A ) , F ( B ) ) und alle ϵ > 0 , δ > 0 st F 1 ( j 0 δ , j 0 + δ ) ( F 1 ( j 0 ) ϵ , F 1 ( j 0 ) + ϵ )

(der Fall wo j 0 = F ( A ) oder F ( B ) ähnelt dem Folgenden und erfordert nur, dass Sie entweder die linke oder die rechte Hälfte der beteiligten Intervalle ignorieren)

Satz γ = Mindest ( ϵ , F 1 ( j 0 ) ( A ) , B F 1 ( j 0 ) ) . Beachten Sie das hier γ ϵ . Es ist leicht zu sehen, dass das Set ( F 1 ( j 0 ) γ , F 1 ( j 0 ) + γ ) besteht in [ A , B ] .

Jetzt bedenke ( F ( F 1 ( j 0 ) γ ) , F ( F 1 ( j 0 ) + γ ) ) . Weil F 1 streng steigend ist, ist leicht zu erkennen, dass dieses Intervall darunter abgebildet wird F 1 hinein ( F 1 ( j 0 ) γ , F 1 ( j 0 ) + γ ) .

Zum Schluss einfach einstellen δ = Mindest ( F ( F 1 ( j 0 ) + γ ) j 0 , j 0 F ( F 1 ( j 0 ) γ ) ] . Der Satz ( j 0 δ , j 0 + δ ) ist eine Teilmenge von ( F ( F 1 ( j 0 ) γ ) , F ( F 1 ( j 0 ) + γ ) ) , und so wird in gesendet ( F 1 ( j 0 ) γ , F 1 ( j 0 ) + γ ) von F 1 .

Weil γ ϵ , das haben wir dann F 1 ( j 0 δ , j 0 + δ ) ( F 1 ( j 0 ) ϵ , F 1 ( j 0 ) + ϵ ) , wie gewünscht.

Können Sie bitte erläutern, was Sie mit "abgebildet unter F 1 ? Ich bin mir nicht ganz sicher, was du da meinst.
@ Shrodinger149 Kennen Sie die Idee des Bildes eines Sets? Wenn ja, wenn ich so etwas sage wie „set A ist unter abgebildet F 1 hinein B “ oder „eingestellt A wird abgebildet von F 1 hinein B ', Ich sage F 1 ( A ) B . Das ist auch das, was ich damit meine, wenn ein Set 'gesendet' wird F 1 (oder F ).
Ich verstehe. Alles sieht gut aus und macht Sinn (obwohl es eine Weile gedauert hat, bis ich mich darum gekümmert habe). Ist das Ihr eigener Ansatz oder haben Sie diesen Beweis schon einmal gesehen? Ich würde wirklich gerne wissen, wie dieser Beweis ausgedacht wird, da er mir nicht intuitiv erscheint (wenn es eine Möglichkeit gibt, darauf zu antworten, weil ich verstehe, dass manchmal Dinge einfach "zu Ihnen kommen"). Danke für die Antwort.
@ Shrodinger149 Ich weiß deine Frage zu schätzen. Dieser Ansatz ist durch mein (ziemlich grundlegendes) Verständnis der Punktmengentopologie motiviert. Einer der Hauptvorteile beim Studium analysebezogener Ergebnisse im Rahmen der Topologie ist ihre Allgemeingültigkeit. Die Definition von Kontinuität, die ich in meiner Antwort verwendet habe, ist ein Sonderfall dieser allgemeinen Definition . Allerdings werden die Dinge, die Sie in einer einführenden Topologie finden, für die Analyse auf einer höheren Ebene nicht wirklich nützlich sein, da Differenzierung/Integration nie berührt werden. (1/2)
@ Shrodinger149 Dies liegt daran, dass dieser Raum bestimmte spezifische Eigenschaften haben muss, um Kalkül (oder etwas "ähnliches" Kalkül) in einem Raum durchzuführen (die einem durchschnittlichen topologischen Raum fehlen). Es gibt einen Ansatz zur Analysis, der sehr allgemein ist und einige Konzepte aus der Topologie verwendet (siehe Bücher wie „Calculus on Manifolds“), aber der größte Teil der Topologie befasst sich nicht mit solchen Dingen. Die Topologie hilft also im Grunde bei Ergebnissen in Bezug auf Kontinuität, Grenzen usw., aber nicht bei der Analyse an sich. Es ist also nicht wirklich ein geeigneter Weg, um ein Verständnis für Analyse zu erlangen (außer vielleicht in einem ersten Kurs). (2/2)